LSAT and Law School Admissions Forum

Get expert LSAT preparation and law school admissions advice from PowerScore Test Preparation.

User avatar
 Dave Killoran
PowerScore Staff
  • PowerScore Staff
  • Posts: 5853
  • Joined: Mar 25, 2011
|
#40892
Complete Question Explanation
(The complete setup for this game can be found here: lsat/viewtopic.php?t=15551)

The correct answer choice is (C)

Answer choice (A): This answer choice violates both the first and fourth rules.

Answer choice (B): This answer choice violates the second rule that prohibits H from being first.

Answer choice (C): This is the correct answer choice.

Answer choice (D): This answer choice violates the third rule that indicates L and I are consecutive.

Answer choice (E): This answer choice violates the last rule that stipulates that F is first or seventh.

Get the most out of your LSAT Prep Plus subscription.

Analyze and track your performance with our Testing and Analytics Package.